what angles are adjacent to 10 ? select all that apply

What Angles Are Adjacent To 10 ? Select All That Apply

Answers

Answer 1

Answer:

15 & 1

Step-by-step explanation:

The angles are adjacent are not yes by tracking
Answer 2
The answer is 15 and 1

Related Questions

Help right now please.

Answers

I solved and the mean is 10

Answer:

4.8

Step-by-step explanation:

Can I get some help plzzz. Also plz give steps

Answers

Answer:

r=2.9 cm

Step-by-step explanation:

Given:

volume=486π cm^3

height=18 cm

volume of a cone=πr^2h

486=π*r^2*18

486=18π*r^2

486/18=πr^2

27/π=r^2

27/3.14=r

8.5987=r^2

[tex]\sqrt{8.5987[/tex] =r

2.9323 =r

2.9=r(after converting to nearest tenth)

R=2.9cm………………………………………

Please help..write expanded form of the expression

Answers

79203782 there u go so that’s should he
79203782 there you go






The solution of the equation 5x+3=3x+5 is
2 or 1

Answers

Answer:

=

1

Step-by-step explanation:

The answer for the solution to you’re problem is 1

which angles are vertical to each other ?

Answers

Answer:

11 and 9

Step-by-step explanation:

Answer:

<12 and <10

Because they are opppsite to each other. So they are vertically opposite angle (V.O.A).

emma gave 2/5 of her money and an additional $9 to olivia. she gave 4/7 of her remaining money and an additional $1 to Ava. emma then had $17 left. how much did emma have at first

Answers

yo wsp how is it going kane
I think she has 85 left, I did the math but ion feel like writing it out. I could be wrong

giving brain list for correct

Answers

Answer:

216 ft^2

Step-by-step explanation:

area of a triangle: (height * base)/2

in this problem,

height = 12, base = 36

area = 12*36/2 = 6*36 = 216

216 ft^2

Answer: 216

Step-by-step explanation:

I need to get aleks done on june 30th

Answers

Answer:

Down

Step-by-step explanation:

2x-8<-24

Down cause that what the person said

which angles are vertical to each other ?
I'll mark brainliest

Answers

Answer:

5 and 7

Step-by-step explanation:

vertical angles are opposite from each other

Answer:

5 and 7

Step-by-step explanation:

The area of the rectangular floor in Tamara's room is 95 5/6 square
feet. The width of the room is 8 1/3 feet. What is the length of Tamara's
room?

Answers

It would be 11 1/2 ft.
Area = Length x Width
95 5/6 = L x 8 1/3
95 5/6 divided by 8 1/3
= 11 1/2 ft

can someone please answer these (real answers only)

Answers

5. The number of visitors increase in April, spike in August and then decrease in September.

6. December is in winter, therefore many people do not travel during December.

7. There are not a lot of holidays in February and March, so people are either working or going to school instead of visiting.

I hope this helped!
“5. The number of visitors increases in April, spike in August and then decreases in September. 6. December is in winter, therefore many people do not travel during December. 7. There are not a lot of holidays in February and March, so people are either working or going to school instead of visiting”

which angle is adjacent to 9 ?

Answers

angle 12 it means next to or closest so it’s in the same line as 12
Angle twelve would be the angle that is adjacent to angle nine

Please help!!!!!!!!!!!!!!!!!!!!

Answers

1. Corresponding

2. Angle 8
the first one is corresponding
the second one is angle 8

ok this is like not helpful but i did this i past my math class
In my class we used to call y=mx+b 'whY is My eX whith the ️!tch' and it worked i passed ;-;
thats all just something to help lol

Answers

Answer:

Step-by-step explanation:

Answer:

Ask this to those who have choose collage level MARK ME AS THE BRAINLIEST

Step-by-step explanation:

Help please!! Will give Brainly, and 50 Points.

Answers

Answer:

c. (11,-21)

Step-by-step explanation:

to visualize this, i put it on a graph and if you graph the 2 lines you find the point they intersect, which is (11, -21) Hope this helps!

Answer:

C

Step-by-step explanation:

Find the radius of the circle.
Circumference = 18.84 yd
a. 3 yd
b. 59.16 yd
c. 6 yd
d. 118.32 yd

what's the answer??​

Answers

Answer:

A. 3yd

Step-by-step explanation:

There are two formulas that can be used for the circumference of a circle:

c = πd and c = 2πr

Since we want to solve for the radius, let's use c = 2πr

The circumference, 'c', is 18.84 inches

Since 'π' is an irrational number, we use its approximation of 3.14

We are trying to solve for 'r', which is the radius

The circumference, 'c', is 18.84 inches

Since 'π' is an irrational number, we use its approximation of 3.14

We are trying to solve for 'r', which is the radius

Plugging in, we get:

18.84 = 2(3.14)r

Since 'π' is an irrational number, we use its approximation of 3.14

Notice there are the same amount of decimal places (two!) in the answer as the combined amount of spaces in the numbers being multiplied!

18.84 = 6.28r

The goal is to isolate and solve for 'r', which is the radius

18.84/6.28 = 3

So we get "3 = r", which means the radius is 3 inches!

I NEED HELP ILL GIVE BRAINLY BUT PLZ EXPLAIN NOT GUESSES

Answers

Bro ,, The values are ,, These are absolute correct answers.

1.45 degrees

2.45 dehrees

3.135 degrees

4.135 degrees

5.45 degrees

6.135 deegres

please give brainliest

Answer:

Step-by-step explanation:

for first fox

let it be x

x + 135 degree = 180 degree (being straight line)

x = 180 - 135

x = 45 degree

therefore figure upper box is 45 degree

for box that is  just below 135 degree

first fox = box just just below 135 degree

45 degree = box just below 135 degree

for box that is just below 45 degree

box that is just below 45 degree = 135 degreee (being vertically opposite angles)

for the box that lies on the left side of the transversal

let the fourth box that les on the left side be x

135 + x = 180 degree (being co interior angle)

x = 180 - 135

x = 45 degree

for the box that lies on the right side of the transversal .Let it be y

45 + y = 180 degree

y = 180 - 45

x = 135 degree

y = last box that lies on the left side of 45 degreeing vetically opposite angles)y

last box = 135 degree

help me please (look at image)

Answers

about 3 inches is the answer

Which value(s) from the set {5,7,9,11,13} make the inequalty w-4<8 true?

A. {5,7,9,11}
B. {11,13}
C. {13}
D. {5,7,9}

Answers

Answer:

5 7 9 11

Step-by-step explanation:it took me a little plz give me brainlist and a thank u

The answer is A I hope this helped

Help with math earn 10 points

Answers

Answer:

2 I think

Step-by-step explanation:

It is 2 (Unnecessary Stuff)

Help with math 10 points

Answers

Answer:

Step-by-step explanation:

(4,5)

the answer is (4,5) :))

Use Net to find surface area of regular pyramid!! Pls help!!!!!!

Answers

Answer:

78

Step-by-step explanation:

The answer is 78 I just did this 3 days ago

Find the height and volume of cone:

Answers

Answer:

v=9

h=15

Step-by-step explanation:

V=9
H=15 I double checked both of them

Which expression has a value of 6?

Answers

Answer:

Option 4

Step-by-step explanation:

The answer is option 4 because whenever a number is enclosed in those two lines that come in the left and right sides it means absolute value and absolute value makes the number positive regardless of the sign that it has, so in this case you have the negative but the absolute value makes it positive since it has the absolute value lines.

Option 4 whenever you see “| |” remember to look for the opposite of what’s in those lines for example:

|-3| the opposite would be positive 3!

Hi this is 7th grade geometry with screenshot

Answers

Answer:

80

Step-by-step explanation:

ㄥAEC+160=180

ㄥAEC=20

ㄥAEC+x+80=180

20+x+80=180

x=80

There is a more simple method

x+80=160

x=80

It’s 80 because it is and it’s right

HELPPPPPP QUICKKKKK!!!!!!!!!!

Answers

Answer:

35x+55 = 160

Step-by-step explanation:

Cost equal flat rate plus rate per hour times hours  where x is the number of hours worked

cost = 55 + 35x

160 = 55 + 35x

Rewriting

35x+55 = 160

D is the correct answer, 35x + 55 =160!

A wall is 60 feet tall. To avoid tipping over, the ladder must be placed 11 feet from the base of the wall. How tall of a ladder must you have to reach all the way to the top of the 60 foot wall?

Answers

61 feet

I got this by doing the pythagorean theorem, which is a^2+b^2=c^2

Plugging in the values, it is 60^2+11^2=c^2

Once solved, it gives you 61 feet
61 feet is the answer okkkkkkkkk

What is the volume of the following rectangular prism?

Answers

Answer:

7/8

Step-by-step explanation:

Volume=AH

A= Area of base

H= Height

So

Volume= 1/3/4*(1/2)

=7/8

Answer:

7/8

Step-by-step explanation:

Cindy's yard is 30 feet long and 40 feet wide. Edward charges 2 cents per square foot to mow yards. How much would Cindy have to pay to have Edward mow her yard?

A.) $24

B.) $240

C.) $48

D.) $480

Answers

The answer is A 24$ because 30x40 is 1200 and 1200x 0.02 is 24
A 24 because u gotta do 30x40

(trigonometry question)
-I'll give branliest as a reward-

Answers

Answer:

AC=2.868

Step-by-step explanation:

using pythagoras theorem

take 35 degree as reference angle

hypotenuse =5 , opposite =AC , adjacent=BC

using sin rule

sin 35=opposite/hypotenuse

0.5735=AC/5

0.5735*5=AC

2.8675=AC

2.868=AC

Ac=2.868 sorry if I am wrong
Other Questions
The CEO/chairman of PharmaPacifica was recently killed in an airplane crash. This tragedy has thrown PharmaPacifica into turmoil as there is no one in the organization qualified to step into the former CEO's shoes. This is an example of: a. the risk inherent in CEO duality. b. excessive reliance on the internal managerial labor market. c. managerial hubris. d. a failure of succession management. 33. What does equal in the solution of the system of equations below? 33 4y - z= 14 2r + 3y - 32 = 17 x + 2y + x = 20 10 04 -10 O-4 Please help me!!!!If you get the answer right, you will get brainliest, 5 stars, and a thanks. Preview the following writing prompt. Then answer the question that follows.Prompt: doctors are learning more about the series ended injuries suffered by people who play certain Sports. What changes should the sports program in high school next to protect students from head injuries?Piece of writing prompt, which research question would best help you narrow your topic for a five paragraph essay (300 words) ?A. When did Doctor's first discover the head injuries occurred in contact Sports?B. Is it easy to recover from a back injury than it is to recover from a head injury?C. What is just absurd coaches take during games and practices to keep students safe?D. How can schools encourage more students to participate in school sports?E. Why doesn't the human skull provide enough protection to avoid head injuries from school? A mother was frustrated with her four-year-old child, who would not stop throwing a tantrum in a large chain department store. After screaming that he wanted a toy over and over again, and trying to run away from her to the toy aisle, the mother swatted her son on the behind. Another woman in the store saw the mother swat the son and saw the child cry and grab his behind; because she believed that spanking was child abuse, she reported the mother to the police. The police arrested the mother for battery. When they examined the child, he had no marks on his body. Will the mother like Suppose that the 100 people in society C all know the same 10 facts, while the 10 people in society D specialize, with each person knowing 5 unique facts as well as 5 facts also known by the other 9 members of the society. If the standard of living is roughly equivalent to the average number of social facts known per person, how many times better is the standard of living in society D Please Hurry!!!!!!!!!!!!!!!!What do the majority of critics consider the main reason that Friedan's book The Feminine Mystique sparked a feminist revolution across America when it was published in 1963?That it didn't advocate women leaving their traditional role.That it was marketed and sold only on college campuses to young women.That it expressed what many women were already feeling.That it was written in easily accessible language. The first step of a competitive strength assessment is to Multiple choice question. determine the company's overall net competitive advantage or disadvantage. conduct a SWOT analysis of the company. make a list of all competing firms in the industry. determine all of the industry factors that contribute to a firm's competitive strength or weakness. Answer please thanks. Brainliest In "The Necklace," what is Madame Loisel's attitude toward the sacrifices that she and her husband must make as they workto replace the necklace?determined and couragon Can some please help me? ~ASAP~ helppppopoop help help help what is the corrct answer If a computer prints 225 lines in 3 seconds how many lines can it print per minute why do we find tea to be tasteless after we have eaten sweet or candies How to shade in General Reread the first paragraph of O.Henry's short story "Hearts and Hands." At Denver there was an influx of passengers into the coaches on the eastbound B. & M. express. In one coach there sat a very pretty young woman dressed in elegant taste and surrounded by all the luxurious comforts of an experienced traveler. Among the newcomers were two young men, one of handsome presence with a bold, frank countenance and manner; the other a ruffled, glum-faced person, heavily built and roughly dressed. The two were handcuffed together. What part of the paragraph helps create the misunderstanding at the center of the story?1. the descriptions of the two men who have just entered the train car2. details about the clothing of the young woman on the train3. the fact that B. & M. express train was headed east from Denver4. the fact that two men were handcuffed together The "L" pine tree cones are attached to the stem; the "S" pinecones are attached toA. a leafB. a stalkC. the trunkD. none of the above In a polar molecule, the partial positive charge on one molecule is attracted toward the partial _____ charge on a nearby molecule. What is the difference between old and new imperialism? Which of the following accurately describes circuits